Elena is making greeting cards, which she will sell by the box at an arts fair. She paid $54 for a booth at the fair, and the materials for each box of cards cost $9. 50. She will sell the cards for $11 per box. At some point, she will sell enough cards so that her sales cover her expenses. How much will the sales and expenses be? How many boxes of cards will that take?

Answers

Answer 1

Elena's total sales and expenses will amount to $54 for booth cost, $9.50 per box for materials, and $11 per box for sales. The number of boxes needed to cover the expenses depends on the calculation of the breakeven point.

Elena's expenses consist of the booth cost and the materials for each box of cards. The booth cost is $54. The materials for each box cost $9.50. To calculate the breakeven point, we need to determine how many boxes Elena needs to sell to cover her expenses.

Let's assume Elena sells X boxes of cards. The total expenses can be calculated as $54 (booth cost) + $9.50 (material cost per box) * X (number of boxes). The total sales will be $11 (selling price per box) * X (number of boxes).

To find the breakeven point, we need the total sales to equal the total expenses. So, we set up the equation: $54 + $9.50X = $11X.

Simplifying the equation, we get $54 = $1.50X.

Dividing both sides by $1.50, we find X = 36.

Therefore, Elena needs to sell 36 boxes of cards to cover her expenses. The total sales will be $11 (selling price per box) * 36 (number of boxes) = $396. The total expenses will be $54 (booth cost) + $9.50 (material cost per box) * 36 (number of boxes) = $342.

Learn more about  breakeven point here :

https://brainly.com/question/27106216

#SPJ11


Related Questions

Use a power series to approximate the value of the integral with an error of less than 0.0001. (Round your answer to five decimal places.)I=∫x ln(x+1)dx.

Answers

To approximate the integral I = ∫x ln(x+1)dx using a power series, we can first use integration by parts to obtain:

I = x(ln(x+1) - 1) + ∫(1 - 1/(x+1))dx

Next, we can use the geometric series expansion to write 1/(x+1) as:

1/(x+1) = ∑(-1)^n x^n for |x| < 1

Substituting this into the integral above and integrating term by term, we get:

I = x(ln(x+1) - 1) - ∑(-1)^n (x^(n+1))/(n+1) + C

where C is the constant of integration.

To obtain an error of less than 0.0001, we need to find a value of n such that the absolute value of the (n+1)th term is less than 0.0001. We can use the ratio test to find this value:

|(x^(n+2))/(n+2)|/|(x^(n+1))/(n+1)| = |x|/(n+2)

For the ratio to be less than 0.0001, we need:

|x|/(n+2) < 0.0001

Choosing x = 0.5, we get:

0.5/(n+2) < 0.0001

Solving for n, we get n > 4980.

Therefore, we can approximate the integral I to within an error of 0.0001 by using the power series:

I ≈ x(ln(x+1) - 1) - ∑(-1)^n (x^(n+1))/(n+1)

with n = 4981.

To know more about power series, visit:

https://brainly.com/question/29896893

#SPJ11

Suppose you have a friend and that friend lives 3 miles away. You ride your bike there but on the way there you forgot you had chores to do. So you continue to ride there to tell your friend that you can't stay and immediately turn back around. Since you're in a hurry, you ride 5 mph faster than your trip to your friend. The total round trip took 30 minutes.

Answers

The chores overshadowed the joy of our Reunion, learned a valuable lesson about managing time and prioritizing obligations

Embarking on a bike ride to visit a friend who lives 3 miles away, little did I know that a looming sense of forgotten chores would soon disrupt my plans. Determined to fulfill my obligations, I mustered the strength to continue riding towards my friend's house, albeit with a heavy heart.

I pedaled towards my destination, the weight of my impending chores grew heavier with each passing moment. Thoughts of unfinished tasks occupied my mind, and I knew that I couldn't stay for long once I reached my friend's place. However, in my haste, a newfound urgency propelled me forward, and I found myself pedaling at a speed 5 mph faster than my initial journey.

With this increased velocity, the return trip promised to be swifter, yet time was slipping away. My mind raced as I calculated the implications of my predicament. The total round trip, comprising both the journey to my friend's house and the hurried return, needed to be accomplished within a tight time frame of 30 minutes.

As I approached my friend's house, I realized that I had no choice but to deliver my news swiftly and immediately turn back around. The momentary joy of reunion would be overshadowed by the pressing chores that awaited me. Regrettably, I bid my friend a hasty farewell, explaining the circumstances that compelled my premature departure.

Once on my bike again, I kicked up the pace, utilizing the extra speed to my advantage. The wind rushed past my face as I hurriedly retraced my path, pushing myself to complete the return trip as swiftly as possible. The seconds ticked away relentlessly, as the pressure mounted to make it back within the allocated timeframe.

In a flurry of determination, I managed to reach home just in the nick of time, fulfilling my duties and responsibilities. Exhausted but relieved, I contemplated the whirlwind of events that had transpired within the span of this half-hour adventure.

To know more about Reunion.

https://brainly.com/question/29611437

#SPJ11

Note the question be like :

A company is hosting a team-building event and has allocated a total of 4 hours for various activities. If Activity A takes 1 hour, Activity B takes 2 hours, and Activity C takes 45 minutes, what is the maximum amount of time that can be dedicated to Activity D while still staying within the allocated 4-hour timeframe?

for all real numbers x, cos2 (3x) sin2 (3x) =

Answers

All real numbers x, cos²(3x) sin²(3x) = sin²(3x)(5 - 4cos²(3x)).



Using the identity cos(2θ) = 1 - 2sin²(θ), we can simplify the expression as follows:

cos²(3x) sin²(3x) = (1 - sin²(6x))(sin²(3x))
= sin²(3x) - sin²(6x)sin²(3x)

Using the identity sin(2θ) = 2sin(θ)cos(θ), we can express sin²(6x) as 4sin²(3x)cos²(3x):

sin²(6x) = (2sin(3x)cos(3x))²
= 4sin²(3x)cos²(3x)

Substituting this expression into our original equation, we get:

cos²(3x) sin²(3x) = sin²(3x) - 4sin²(3x)cos²(3x)sin²(3x)
= sin²(3x)(1 - 4cos²(3x))

Using the identity cos(2θ) = 1 - 2sin²(θ) again, we can express 4cos²(3x) as 2(2cos²(3x) - 1):

cos²(3x) sin²(3x) = sin²(3x)(1 - 2(2cos²(3x) - 1))
= sin²(3x)(5 - 4cos²(3x))

Therefore, for all real numbers x, cos²(3x) sin²(3x) = sin²(3x)(5 - 4cos²(3x))

Learn more about real numbers

brainly.com/question/551408

#SPJ11

Use Green's theorem for circulation to evaluate the line integral θ∫θ F. dr. F = ((xy^2 + 2x), (3x + y^2)) and C is the positively oriented boundary curve of the region bounded by y = 1, y = 2 y = -2x, and x = y^2 2(3√ 2 +2)

Answers

Answer:

The value of the line integral is 2(3√2 + 2).

Step-by-step explanation:

We can use Green's theorem for circulation to evaluate the line integral:

θ∫θ F · dr = ∬R ( ∂Q/∂x - ∂P/∂y ) dA

where F = (P, Q), R is the region bounded by the curve C, and the integral is over R.

First, we need to find the partial derivatives of P and Q:

∂P/∂y = 0

∂Q/∂x = y^2 + 2

Then, we can evaluate the double integral over the region R:

θ∫θ F · dr = ∫-2^(1/2)^(3/2) ∫y^2/2 -2x (y^2 + 2) dx dy

Evaluating the inner integral with respect to x, we get:

∫y^2/2 -2x (y^2 + 2) dx = (y^4/8 - y^2 - 2xy^2 - 4x)|y^2/2 -2x = (-9/8)y^2 - 8y^(5/2)/5

Then, evaluating the outer integral with respect to y, we get:

θ∫θ F · dr = ∫-2^(1/2)^(3/2) (-9/8)y^2 - 8y^(5/2)/5 dy

= (-9/24)(y^3)|-2^(1/2)^(3/2) - (8/7)(y^(7/2))|-2^(1/2)^(3/2)

= 2(3√2 + 2)

Therefore, the value of the line integral is 2(3√2 + 2).

To know more about Green's theorem refer here

https://brainly.com/question/30763441#

#SPJ11

Let Y1, Y2,...,Yn denote a random sample from a population with mean µ and variance s^2. Consider the following three estimators for µ:
µ^1 = .5(Y1 + Y2), µ^2 = .25(Y1) + [Y2 + ... + Yn-1 / 2(n-2)] + .25Yn, µ^3 = Y bar.
a) Show that each of the three estimators is unbiased.
b) Find the efficiency of µ^3 relative to µ^2 and µ^1, respectively.

Answers

The efficiency of µ^3 is [(n-2)^2]/(2n-1) relative to µ^2, and 2s^2/n relative to µ^1.

To show that each of the three estimators is unbiased, we need to show that their expected values are equal to µ, the true population mean.

For µ^1: E(µ^1) = E[.5(Y1+Y2)] = .5E(Y1) + .5E(Y2) = .5µ + .5µ = µ

For µ^2: E(µ^2) = E[.25Y1 + (Y2+...+Yn-1)/2(n-2) + .25Yn] = .25E(Y1) + (n-2)/2(n-2)E(Y2+...+Yn-1) + .25E(Yn) = .25µ + .75µ + .25µ = µ

For µ^3: E(µ^3) = E(Y bar) = µ, since Y bar is an unbiased estimator of µ.

Therefore, all three estimators are unbiased.

The efficiency of µ^3 relative to µ^2 is given by:

efficiency of µ^3/µ^2 = [(Var(µ^2))/(Var(µ^3))] x [(1/n)/(1/2(n-2))]^2

To find Var(µ^2), we can use the formula for the variance of a sample mean:

Var(µ^2) = Var(.25Y1) + Var[(Y2+...+Yn-1)/2(n-2)] + Var(.25Yn)

Since all Y's are independent and have the same variance s^2, we get:

Var(µ^2) = .25^2Var(Y1) + [1/(2(n-2))]^2(n-2)Var(Y) + .25^2Var(Yn) = s^2/4 + s^2/2(n-2) + s^2/4 = s^2/2(n-2) + s^2/2

Similarly, we can find Var(µ^3) = s^2/n.

Plugging these values into the efficiency formula, we get:

efficiency of µ^3/µ^2 = [s^2/(2(n-2) + n)] x [(2(n-2))/n]^2 = [(2(n-2))^2]/(2n(n-2)+n) = [(n-2)^2]/(2n-1)

The efficiency of µ^3 relative to µ^1 is given by:

efficiency of µ^3/µ^1 = [(Var(µ^1))/(Var(µ^3))] x [(2/n)/(1/n)]^2 = [s^2/(2n)] x 4 = 2s^2/n

To learn more about Efficiency :

https://brainly.com/question/15418098

#SPJ11

A large tree, b, removes 1. 5 kg of pollution from the air each year. A small tree, s,


removes 0. 04 kg of pollution each year. An urban forest has 1,650 large and small trees.


Together, these trees remove 1,818 kg of pollution each year. Which system of equations


could be used to find the number of large and small trees in the forest?

Answers

Let's denote the number of large trees as 'L' and the number of small trees as 'S'.

According to the given information, a large tree removes 1.5 kg of pollution per year, and a small tree removes 0.04 kg of pollution per year. The total pollution removed by all the trees in the forest is 1,818 kg per year.

We can set up the following system of equations:

Equation 1: L + S = 1,650 (since the total number of trees in the forest is 1,650)

Equation 2: 1.5L + 0.04S = 1,818 (since the total pollution removed by the trees is 1,818 kg per year)

These two equations can be used to find the number of large trees (L) and small trees (S) in the forest.

Learn more about  equations here:

https://brainly.com/question/29538993

#SPJ11

let u={12,13, 14,15,16 ,17,18} and a={12, 14, 15, 18}. find a the roster method to write the set A'. A' = (Use a comma to separate answers as needed.)

Answers


The roster method to write the set A' as {13, 16, 17}.

- "Set" is a collection of distinct objects, which can be numbers or other elements.
- "Roster method" is a way of listing all the elements in a set by separating them with commas and enclosing them within braces { }.
Now, let's find set A', which is the complement of set A with respect to set U. This means that A' contains all the elements in U that are not in A.
U = {12, 13, 14, 15, 16, 17, 18}
A = {12, 14, 15, 18}
To find A', we will list the elements from set U that are not in set A:
A' = {13, 16, 17}
So, using the roster method, the complement of set A (A') is written as:
A' = {13, 16, 17}

In summary, the roster method is useful for listing all the elements in a set. By finding the complement of set A with respect to set U, we can use the roster method to write the set A' as {13, 16, 17}.

Learn more about Set here:

https://brainly.com/question/8053622

#SPJ11

True/False
- If the obtained F value = .77 and the critical F value = 3.40, the researcher would reject the null hypothesis.
- The F-test is the ratio of the variance within groups over the variance between groups.
- If a researcher has found the F statistic is significant they must then conduct an eta-squared test to be able to report which groups means are significantly different from other group means.
- ANOVAs are useful for independent variables that have more than two values because this test assumes that the samples means are independent.
- In ANOVA, it is possible to have negative values for the sums of squares and the mean squares.

Answers


1. If the obtained F value = .77 and the critical F value = 3.40, the researcher would reject the null hypothesis.
False. The obtained F value is less than the critical F value, so the researcher would fail to reject the null hypothesis.

2. The F-test is the ratio of the variance within groups over the variance between groups.
False. The F-test is the ratio of the variance between groups over the variance within groups.

3. If a researcher has found the F statistic is significant, they must then conduct an eta-squared test to be able to report which groups means are significantly different from other group means.
False. If the F statistic is significant, the researcher would conduct post-hoc tests (e.g., Tukey's HSD or Bonferroni) to determine which group means are significantly different, not an eta-squared test.

4. ANOVAs are useful for independent variables that have more than two values because this test assumes that the samples means are independent.
True. ANOVAs are designed to analyze the differences among group means in a sample, making them suitable for independent variables with more than two values.

5. In ANOVA, it is possible to have negative values for the sums of squares and the mean squares.
False. In ANOVA, sums of squares and mean squares are calculated using squared values, so they cannot be negative.

1) In hypothesis testing using ANOVA, the obtained F value is compared to the critical F value to determine whether the null hypothesis should be rejected or not. If the obtained F value is greater than the critical F value, then the researcher would reject the null hypothesis and conclude that there is a significant difference among the group means. However, if the obtained F value is less than the critical F value, then the researcher would fail to reject the null hypothesis and conclude that there is no significant difference among the group means. Therefore, in this scenario, the researcher would fail to reject the null hypothesis.

2) The F-test in ANOVA is used to compare the variance between groups to the variance within groups. The formula for the F-test is:

F = variance between groups / variance within groups

Therefore, the F-test is the ratio of the variance between groups over the variance within groups, not the other way around.

3) If the F statistic is significant, it means that there is a significant difference among the group means. However, the F test does not tell us which group means are significantly different from each other. To determine which group means are significantly different, the researcher would conduct post-hoc tests such as Tukey's HSD or Bonferroni. The eta-squared test is used to measure the effect size of the independent variable on the dependent variable, but it is not used to determine which group means are significantly different.

4) ANOVA (Analysis of Variance) is a statistical method used to test for significant differences among the means of two or more independent groups. ANOVA is a suitable test for independent variables that have more than two values because it can analyze the differences among multiple group means simultaneously.

5) In ANOVA, the total sum of squares (SST), the sum of squares between groups (SSB), and the sum of squares within groups (SSW) are calculated. The mean square between groups (MSB) and the mean square within groups (MSW) are then calculated by dividing the SSB and SSW by their respective degrees of freedom. Since all of these calculations involve squared values, the sums of squares and mean squares cannot be negative.

Learn more about ANOVA :

https://brainly.com/question/31809956

#SPJ11

Help out asap, fairly easy question, Algebra 1

Answers

The equation for the exponential function in the table of ordered pairs (x, y) is; y = 3·6ˣ

What is a form of an exponential function equation?

An exponential function or equation can be presented as follows;

y = a·bˣ, where; x is the input  variable and y is the value of the function.

The values in the table of the ordered pair indicates;

(-1, 1/2), (0, 3), (1, 18)

1/2 = a·b^(-1)

3 = a·b^(0) = a

a = 3

1/2 = 3·b^(-1) = 3/b

b = 3/(1/2) = 6

The possible exponential function is; y = 3·6ˣ

Learn more on exponential functions here: https://brainly.com/question/15662160

#SPJ1

Use the first eight rules of inference to derive the conclusions of the following symbolized arguments:
1. (M ∨ N) ⊃ (F ⊃ G)
2. D ⊃ ∼C
3. ∼ C ⊃ B
4. M • H
5. D ∨ F / B ∨ G

Answers

The conclusion of the argument is B ∨ G.

To derive the conclusion B ∨ G, we can use the rules of inference step by step:

(M ∨ N) ⊃ (F ⊃ G) (Premise)

D ⊃ ∼C (Premise)

∼C ⊃ B (Premise)

M • H (Premise)

D ∨ F (Premise)

M ∨ N (Disjunction Elimination from premise 4)

F ⊃ G (Modus Ponens using premises 1 and 6)

∼C (Modus Ponens using premises 2 and 4)

B (Modus Ponens using premises 3 and 8)

D (Disjunction Elimination from premise 5)

F (Disjunction Elimination from premise 5)

G (Modus Ponens using premises 7 and 11)

B ∨ G (Disjunction Introduction using conclusion 9 and 12)

Therefore, the conclusion of the argument is B ∨ G.

For more questions like Argument click the link below:

https://brainly.com/question/2645376

#SPJ11

ILL GIVE BRAINLIEST!!!



Two input-output pairs for function f(x) are (−6,52) and (−1,172). Two input-output pairs for function g(x) are (2,133) and (6,−1). Paige says that function f(x) has a steeper slope. Formulate each function to assess and explain whether Paige's statement is correct. (4 points)

Answers

To assess whether Paige's statement is correct about the functions f(x) and g(x) having different slopes, we need to formulate the equations for each function using the given input-output pairs.

To formulate the equations for the functions, we use the slope-intercept form of a linear equation, y = mx + b, where m represents the slope.

For function f(x), we can use the input-output pairs (-6, 52) and (-1, 172). To find the slope, we calculate (change in y) / (change in x) using the two pairs:

m = (172 - 52) / (-1 - (-6)) = 120 / 5 = 24.

So the equation for function f(x) is f(x) = 24x + b.

For function g(x), we use the input-output pairs (2, 133) and (6, -1):

m = (-1 - 133) / (6 - 2) = -134 / 4 = -33.5.

The equation for function g(x) is g(x) = -33.5x + b.

Comparing the slopes, we see that the slope of function f(x) is 24, while the slope of function g(x) is -33.5. Since the absolute value of -33.5 is greater than 24, we can conclude that function g(x) has a steeper slope than function f(x).

Therefore, Paige's statement is incorrect. Function g(x) has a steeper slope than function f(x).

Learn more about Paige's here:

https://brainly.com/question/6871033

#SPJ11

Need Help!

A builder is using the scale drawing shown to build a house.


If the owner decides to increase the living room dimensions by 20%, what is the new length and width of the living room floor?


A: 14.4 feet × 9.6 feet

B: 12.8 feet × 8.4 feet

C: 13.2 feet × 8.8 feet

D: 15.2 feet × 9.8 feet

Answers

The new length and width of the living room floor after increasing the dimensions by 20% are 14.4 feet by 9.6 feet. Option (A) is correct.

Understanding How to Scale Dimension

Let us get the original length and width of the living room. Using the scale of 1 cm = 4 ft, we can convert it to feet:

Original length = 3 cm * 4 ft/cm = 12 ft

Original width = 2 cm * 4 ft/cm = 8 ft

To increase the dimensions of the living room by 20%, we can calculate the increase in length and width:

Increase in length = 20% of 12 ft = 0.2 * 12 ft = 2.4 ft

Increase in width = 20% of 8 ft = 0.2 * 8 ft = 1.6 ft

Adding the increase to the original dimensions, we get the new length and width:

New length = Original length + Increase in length

                   = 12 ft + 2.4 ft = 14.4 ft

New width = Original width + Increase in width

                  = 8 ft + 1.6 ft = 9.6 ft

Therefore, the new length and width of the living room floor after increasing the dimensions by 20% are approximately 14.4 feet by 9.6 feet.

Learn more about scale here:

https://brainly.com/question/31380681

#SPJ1

use the integral test to determine whether the sum converges. [infinity] n = 1 1 n 9 evaluate the following integral. [infinity] 1 x 9 dx 1

Answers

The sum ∑ from n = 1 to infinity of 1/n^9 converges.

We will use the integral test to determine whether the sum converges.

To use the integral test, we need to evaluate the following integral:

∫ from 1 to infinity of 1/x^9 dx

We can integrate this using the power rule of integration:

= [-1/(8x^8)] from 1 to infinity

= [-1/(8 x infinity^8)] - [-1/(8 x 1^8)]

= 0 + 1/8

= 1/8

So, the integral converges to 1/8.

According to the integral test, if the integral converges, then the sum also converges. If the integral diverges, then the sum also diverges. Since the integral converges to a finite value of 1/8, the sum also converges.

The sum ∑ from n = 1 to infinity of 1/n^9 converges.

To know more about sum, visit;

https://brainly.com/question/24205483

#SPJ11

The point P(3, 0.666666666666667) lies on the curve y = 2/x. If Q is the point (x, 2/x), find the slope of the secant line PQ for the following values of x. If x = 3.1, the slope of PQ is: and if x = 3.01, the slope of PQ is: and if x = 2.9, the slope of PQ is: and if x = 2.99, the slope of PQ is: Based on the above results, guess the slope of the tangent line to the curve at P(3, 0.666666666666667).

Answers

The tangent  to the curve at P(3, 0.6666666666667) is -2/ 9 or simply, the tangent  is vertical.

To find the slope of the segment PQ, we must use the formula:

Slope of PQ = (change in y) / (change in x) = (yQ - yP) / (xQ - xP)

where P is the point (3, 0.666666666666667) and Q is the point (x, 2/x).

If x = 3.1, then Q is the point (3.1, 2/3.1) and the slope of PQ is:

Slope of PQ = (2/3.1 - 0.666666666666667) / (3.1 - 3) ≈ -2.623

If x = 3.01, then Q is the point (3.01, 2/3.01) and the slope of PQ is:

Slope of PQ = (2/3.01 - 0.666666666666667) / (3.01 - 3) ≈ -26.23

If x = 2.9, then Q is the point (2.9, 2/2.9) and the slope of PQ is:

Slope of PQ = (2/2.9 - 0.666666666666667) / (2.9 - 3) ≈ 2.623

If x = 2.99, then Q is the point (2.99, 2/2.99) and the slope of PQ is:

Slope of PQ = (2/2.99 - 0.666666666666667) / (2.99 - 3) ≈ 26.23

We notice that as x approaches 3, the slope (in absolute terms) of PQ increases. This suggests that the slope of the tangent  to the curve at P(3, 0.666666666666667) is infinite or does not exist.

To confirm this, we can take the derivative  y = 2/x:

y' = -2/x^2

and evaluate it at x = 3:

y'(3) = -2/3^2 = -2/9

Since the slope of the tangent  is the limit of the slope of the intercept as the distance between the two points approaches zero, and the slope of the intercept increases to infinity as  point Q approaches point P along the curve, we can conclude that the slope of the tangent  to the curve at P(3, 0.6666666666667) is -2/ 9 or simply, the tangent  is vertical.

To know more about slope of the segment refer to

https://brainly.com/question/22636577

#SPJ11

in finding 90 nd 95onfidence intervals for a random sample of 30 students' gpas, one interval was (2.55, 3.05) and the other was (2.60, 3.00).. How would a 99% interval compare? Would it be narrower than both, wider than both, or between the two inwidth? Explain. b. If we wanted to use a 99% confidence level and get a narrower width, how could we change our data collection? a. Choose the correct answer below. OA A 99% interval would be narrower than both-the value oft for a 99% interval is less than both that for a 90% interval and that for a 98% interval OB. A 99% interval would be wider than a 95% intorſal and narrower than a 90% interval—the value oft* for a 99% interval is less than that of a 90% interval but greater than that of a 95% Interval OC. A 99% interval would be wider than a 90% interval and narrower than a 95% intervalho value of t* for a 99% interval is greater than that of a 90% interval but less than that of a 95% interval OD. A 99% interval would be wider than both-the value of t for a 99% interval is greater than both that for a 90% interval and that for a 95% interval b. Choose the correct answer below 13 O A Increase the value of B. Manually reduce the sample standard deviation OC. Increase the number of observations by an appropriate amount OD. Decrease the number of observations by an appropriate amount

Answers

The 99% interval would be wider than a 90% interval and narrower than a 95% interval  and by increasing the number of observations by an appropriate amount we can obtain a narrower width of confidence level.

a. The correct answer is C. A 99% interval would be wider than a 90% interval and narrower than a 95% interval—the value of t* for a 99% interval is greater than that of a 90% interval but less than that of a 95% interval.

This is because as the confidence level increases, the interval width increases as well.

Since a 99% interval requires a larger t-value than a 90% interval, it will be wider.

However, since a 95% interval is wider than a 90% interval, but requires a smaller t-value than a 99% interval, the 99% interval will be narrower than the 95% interval but wider than the 90% interval.

b. The correct answer is: C. Increase the number of observations by an appropriate amount.

To obtain a narrower interval at a higher confidence level, firstly we need to increase the sample size.

This is because a larger sample size reduces the standard error of the mean, which leads to a narrower interval.

Therefore, increasing the number of observations by an appropriate amount is the best way to achieve this.

Learn more about confidence level:https://brainly.com/question/15712887

#SPJ11

Solve 2 of 6:2 in maths and explain the formula you used to solve your maths

Answers

2 of 6 can be written as 2/6 or simplified as 1/3, which means that two parts out of six parts represent one-third of the whole.

To solve 2 of 6, we need to understand the basic concepts of fractions and division.

The formula that we can use to solve 2 of 6 is: 2/6 = 1/3

Fraction is a numerical value that represents a part of the whole.

A fraction consists of two parts: the numerator and the denominator.

The numerator is the number above the fraction line, and

the denominator is the number below the fraction line.

For example, in 2/6, 2 is the numerator, and 6 is the denominator.

To solve 2 of 6, we need to divide 2 by 6.

In other words, we need to find out how many parts of the whole 2 represents out of 6 equal parts.

The formula to divide fractions is:

a/b ÷ c/d = ad / bc.

To solve 2 of 6, we can rewrite it as 2/6 ÷ 1/1.

Then we can use the formula as follows:

2/6 ÷ 1/1 = 2/6 × 1/1 = 2/6

Therefore, 2 of 6 can be written as 2/6 or simplified as 1/3, which means that two parts out of six parts represent one-third of the whole.

To know more about fractions visit:

https://brainly.com/question/10354322

#SPJ11

6(5x8)+6(5-9)+87 all 6th grader who are working on order of operation use this problem as practice

Answers

The expression 6(5x8)+6(5-9)+87 is used by 6th graders as practice for order of operations. The answer to the expression is determined by following the order of operations, which involves evaluating parentheses, performing multiplication and division from left to right, and finally performing addition and subtraction from left to right.

To solve the expression 6(5x8)+6(5-9)+87, we need to follow the order of operations.

First, we evaluate the parentheses:

5x8 = 40

5-9 = -4

Next, we perform multiplication and division from left to right:

6(40) = 240

6(-4) = -24

Finally, we perform addition and subtraction from left to right:

240 + (-24) = 216

So, the answer to the expression is 216.

By practicing problems like these, 6th graders reinforce their understanding of the order of operations and learn how to correctly evaluate expressions involving multiple operations.

Learn more about parentheses here:

https://brainly.com/question/15100042

#SPJ11

Mark, Jessica, and Nate each downloaded music from the same website. Mark downloaded 10 songs in total consisting of pop, rock, and hip hop. Jessica downloaded five times as many pop songs, twice as many rock songs, and three times as many hip hop songs as Mark. She downloaded 28 songs total. Nate downloaded 20 songs total with three times as many pop songs, three times as many rock songs, and the same number of hip hop songs as Mark. Which system of equations represents their music choices? x y z = 10 5x 2y 3z = 28 3x 3y z = 20 x y z = 10 2x 5y 3z = 28 3x 3y z = 20 x y z = 10 5x 2y 3z = 28 3x 3y 3z = 20 x y z = 10 2x 3y 5z = 28 x 3y 3z = 20.

Answers

Thus, the answer is the fourth option which is, x y z = 10 5x 2y 3z = 28 3x 3y 3z = 20.

Mark, Jessica, and Nate each downloaded music from the same website and this music consists of pop, rock, and hip hop songs.

Mark downloaded a total of 10 songs in total, with a combination of pop, rock, and hip hop songs.

Jessica downloaded five times as many pop songs, twice as many rock songs, and three times as many hip hop songs as Mark, with a total of 28 songs.

Nate downloaded 20 songs in total with three times as many pop songs, three times as many rock songs, and the same number of hip hop songs as Mark.

The system of equations that represents their music choices are:

x + y + z = 10

Equation 1 - 5x + 2y + 3z = 28

Equation 2 - 3x + 3y + z = 20

Equation 3 -Let x be the number of pop songs that Mark downloaded.

Let y be the number of rock songs that Mark downloaded.

Let z be the number of hip hop songs that Mark downloaded.

From the given information, Mark downloaded a total of 10

songs, so: x + y + z = 10 Equation 1 Jessica downloaded five times as many pop songs, twice as many rock songs, and three times as many hip hop songs as Mark.

She downloaded 28 songs total, so:

5x + 2y + 3z = 28

Equation 2 Nate downloaded 20 songs in total with three times as many pop songs, three times as many rock songs, and the same number of hip hop songs as Mark,

so: 3x + 3y + z = 20 Equation 3

Therefore, the system of equations that represents their music choices are:

x + y + z = 10

5x + 2y + 3z = 28

3x + 3y + z = 20

To know more about equations visit

https://brainly.com/question/29657983

#SPJ11

If a 0. 5 liter solution of bichloride contains 1 gram of bichloride, then 250ml will contain how many grams of bichloride? *​

Answers

We can set up a proportion to find the number of grams of bichloride in 250 mL:

(1 gram) / (0.5 liter) = (x grams) / (0.25 liter)

Cross-multiplying:

0.5x = 0.25

Dividing both sides by 0.5:

x = 0.25 / 0.5 = 0.5

Therefore, 250 mL will contain 0.5 grams of bichloride.

#SPJ11

Let Gle) be the generating function for the sequence , 3.. Expres the generating ao, a1, a2, a3,.... Express the generating function of each sequence below in terms of r and G(x). (a) 2ao, 2a1,2a2,2a3, .. (b) 0,ao,a1,a2,. (c) 0,0,a2, a3,a4,as. (d) ao, 2a1,4a2,8a3,... (e) ao, a1 +ao, a2 + a1,a3 a2,.
Previous question

Answers

The generating function for the sequence can be expressed as G(x) = 1/(1 - 3x).

How can we express the generating functions of different sequences in terms of r and G(x)?

The generating function G(x) represents a sequence of numbers, where G(x) = a0 + a1x + a2x^2 + a3x^3 + ..., where ai represents the ith term of the sequence.

Step 1: For the given sequence with the generating function G(x) = 1/(1 - 3x), we can express the generating functions of different sequences as follows:

(a) The generating function for the sequence 2ao, 2a1, 2a2, 2a3, ... can be expressed as 2G(x).

(b) The generating function for the sequence 0, ao, a1, a2, ... can be expressed as xG(x).

(c) The generating function for the sequence 0, 0, a2, a3, a4, ... can be expressed as x^2G(x).

(d) The generating function for the sequence ao, 2a1, 4a2, 8a3, ... can be expressed as G(2x).

(e) The generating function for the sequence ao, a1 + ao, a2 + a1, a3 + a2, ... can be expressed as G(x)/(1 - x).

Step 2: How can we express the generating functions of different sequences using the generating function G(x)?

Step 3: The generating function G(x) = 1/(1 - 3x) represents a sequence where the coefficients of the terms correspond to the powers of x. By manipulating the given generating function, we can express the generating functions of different sequences.

For example, to express the generating function of the sequence 2ao, 2a1, 2a2, 2a3, ..., we simply multiply the original generating function G(x) by 2. Similarly, by multiplying G(x) by x, x^2, or 2x, we can obtain the generating functions for the sequences in parts (b), (c), and (d), respectively.

In part (e), the generating function represents a sequence where each term is the sum of the corresponding term and the previous term from the original sequence. To achieve this, we divide G(x) by (1 - x).

Learn more about function

brainly.com/question/30721594

#SPJ11

5-8. The Following Travel Times Were Measured For Vehicles Traversing A 2,000 Ft Segment Of An Arterial: Vehicle Travel Time (s) 40. 5 44. 2 41. 7 47. 3 46. 5 41. 9 43. 0 47. 0 42. 6 43. 3 4 10 Determine The Time Mean Speed (TMS) And Space Mean Speed (SMS) For These Vehicles

Answers

The term ‘arterial’ is used to describe roads and streets which connect to the highways. These roads are designed to help people move around easily and quickly. The study of arterial roads is an important area of transportation engineering.

To calculate the Time Mean Speed (TMS), first, the total distance covered by the vehicles needs to be calculated. Here, the distance covered by the vehicles is 2000 ft or 0.38 miles (1 mile = 5280 ft).Next, the total travel time for all vehicles is calculated as follows:40.5 + 44.2 + 41.7 + 47.3 + 46.5 + 41.9 + 43.0 + 47.0 + 42.6 + 43.3 = 437.0 secondsNow, the time mean speed (TMS) can be calculated as follows:TMS = Total Distance / Total Time = 0.38 miles / (437.0 seconds / 3600 seconds) = 24.79 mphThe Space Mean Speed (SMS) can be calculated by dividing the length of the segment by the average travel time of vehicles. Here, the length of the segment is 2000 ft or 0.38 miles (1 mile = 5280 ft).

The average travel time can be calculated as follows: Average Travel Time = (40.5 + 44.2 + 41.7 + 47.3 + 46.5 + 41.9 + 43.0 + 47.0 + 42.6 + 43.3) / 10= 43.7 seconds Now, the Space Mean Speed (SMS) can be calculated as follows: SMS = Segment Length / Average Travel Time= 0.38 miles / (43.7 seconds / 3600 seconds) = 19.54 mp h Therefore, the Time Mean Speed (TMS) and Space Mean Speed (SMS) for these vehicles are 24.79 mph and 19.54 mph respectively.

TO know more about area visit:

brainly.com/question/30307509

#SPJ11

Let C be the boundary-curve of a 5 x 3 rectangle in the sy-plane, equipped with the counterclockwise orientation. Let F(x,y) = (2y - en *)i +9aj. Use Green's theorem to compute fF.dr.

Answers

The line integral is zero.

What is the result of the line integral using Green's Theorem?

To use Green's Theorem, we need to calculate the curl of the vector field [tex]F(x, y) = (2y - e^{(n*)})i + 9aj[/tex]. The curl of a vector field F = (P, Q) is given by the formula:

curl(F) = (∂Q/∂x - ∂P/∂y)k,

where k is the unit vector in the z-direction.

Let's calculate the curl of F(x, y):

[tex]P = 2y - e^{(n*)}[/tex]

Q = 9a

∂Q/∂x = 0 (since Q does not depend on x)

∂P/∂y = 2

Therefore, the curl of F is:

curl(F) = (∂Q/∂x - ∂P/∂y)k = -2k.

Now, we can apply Green's Theorem. Green's Theorem states that for a vector field F = (P, Q) and a curve C equipped with the counterclockwise orientation,

∫ C F.dr = ∬ R curl(F).n dA,

where n is the unit outward normal vector to the region R enclosed by the curve C.

In this case, the curve C is the boundary of a 5 x 3 rectangle in the sy-plane, equipped with the counterclockwise orientation. The region R is the entire rectangular region.

Since the curl of F is -2k, the dot product of curl(F) with the outward normal vector n will be zero, as k is perpendicular to n.

Therefore, ∬ R curl(F).n dA = 0, and as a result:

∫ C F.dr = 0.

Hence, the value of the line integral ∫ C F.dr using Green's Theorem is zero.

Learn more about Green's Theorem

brainly.com/question/30080556

#SPJ11

Refer to the table on air travel outside of the airport suppose a flight that arrives in el centro is just looking at random what is the password that i did not arrive on time write your answer in love as a fraction decimal and percent explain your reasoning

Answers

The answer as a fraction, decimal, and percent is 3/10, 0.3, and 30%, respectively.

The table on air travel outside of the airport is not provided in the question. However, to answer the question, we can assume that the table contains information about flight arrivals and departure times.In order to determine if a flight arrived on time, we need to know the scheduled arrival time and the actual arrival time. If the actual arrival time is later than the scheduled arrival time, then the flight is considered delayed. If the actual arrival time is earlier than the scheduled arrival time, then the flight is considered early. If the actual arrival time is the same as the scheduled arrival time, then the flight is considered on time.To find the percentage of flights that arrive on time, we need to divide the number of on-time flights by the total number of flights and then multiply by 100. For example, if there are 200 flights and 140 of them arrived on time, then the percentage of flights that arrived on time would be:

(140/200) x 100 = 70%

To find the percentage of flights that did not arrive on time, we need to subtract the percentage of on-time flights from 100. For example, if the percentage of on-time flights is 70%, then the percentage of flights that did not arrive on time would be:

100 - 70 = 30%

Therefore, the answer as a fraction, decimal, and percent is 3/10, 0.3, and 30%, respectively.

To know more about fraction visit:

https://brainly.com/question/10354322

#SPJ11

Given a group of students: G = {Allen, Brenda, Chad, Dorothy, Eric) or G = {A, B, C, D, E, list and count the differen ways of choosing the following officers or representatives for student congress (Allen, Chad, and Eric are men) Assume that no one can hold more than one office. 1) A president, a secretary, and a treasurer, if the president must be a woman and the other two must be men A) BAC, BAE, BCE, DAC, DAE, DCE, BCA, BEA, BEC, DCA, DEA, DEC:12 ways B) CAB, EAB, ECB, CAD, EAD, ECD, ACB, AEB, CEB, ACD, AED, CED; 12 ways C) BAC, BAE, DAC, DAE; 4 ways D) BAC, BAE, BCE, DAC, DAE, DCE 6 ways

Answers

The different ways of choosing a president, a secretary, and a treasurer, with the president being a woman and the other two being men, are 12 ways (option A).


How many ways can the president, secretary, and treasurer be chosen if the president must be a woman and the other two positions must be held by men?

To choose a president, a secretary, and a treasurer from the group of students (G = {Allen, Brenda, Chad, Dorothy, Eric}), with the condition that the president must be a woman and the other two must be men, we can list and count the different ways as follows:

A) The president is Brenda (B), and the two men are Allen (A) and Chad (C): BAC

The president is Brenda (B), and the two men are Allen (A) and Eric (E): BAE

The president is Brenda (B), and the two men are Chad (C) and Eric (E): BCE

The president is Dorothy (D), and the two men are Allen (A) and Chad (C): DAC

The president is Dorothy (D), and the two men are Allen (A) and Eric (E): DAE

The president is Dorothy (D), and the two men are Chad (C) and Eric (E): DCE

The total number of ways: 12

B) The president is Chad (C), and the two men are Allen (A) and Brenda (B): CAB

The president is Eric (E), and the two men are Allen (A) and Brenda (B): EAB

The president is Eric (E), and the two men are Chad (C) and Brenda (B): ECB

The president is Chad (C), and the two men are Allen (A) and Dorothy (D): CAD

The president is Eric (E), and the two men are Allen (A) and Dorothy (D): EAD

The president is Eric (E), and the two men are Chad (C) and Dorothy (D): ECD

The total number of ways: 12

C) The president is Brenda (B), and the two men are Allen (A) and Chad (C): BAC

The president is Brenda (B), and the two men are Allen (A) and Eric (E): BAE

The president is Dorothy (D), and the two men are Allen (A) and Chad (C): DAC

The president is Dorothy (D), and the two men are Allen (A) and Eric (E): DAE

The total number of ways: 4

D) The president is Brenda (B), and the two men are Allen (A) and Chad (C): BAC

The president is Brenda (B), and the two men are Allen (A) and Eric (E): BAE

The president is Brenda (B), and the two men are Chad (C) and Eric (E): BCE

The president is Dorothy (D), and the two men are Allen (A) and Chad (C): DAC

The president is Dorothy (D), and the two men are Allen (A) and Eric (E): DAE

The president is Dorothy (D), and the two men are Chad (C) and Eric (E): DCE

The total number of ways: 6

In summary, there are 12 ways in options A and B, 4 ways in option C, and 6 ways in option D to choose a president, a secretary, and a treasurer with the given conditions.

Learn more about treasurer
brainly.com/question/21709999
#SPJ11

Let f(x)=x2 2x 3. What is the average rate of change for the quadratic function from x=−2 to x = 5?.

Answers

The average rate of change is the slope of a straight line that connects two distinct points.

For instance, if you are given a quadratic function, you will need to compute the slope of a line that connects two points on the function’s graph. What is a quadratic function? A quadratic function is one of the various functions that are analyzed in mathematics. In this type of function, the highest power of the variable is two (x²). A quadratic function's general form is f(x) = ax² + bx + c, where a, b, and c are constants. What is the average rate of change of a quadratic function? The average rate of change of a quadratic function is the slope of a line that connects two distinct points. To find the average rate of change, you will need to use the slope formula or rise-over-run method. For example, let's consider the following function:f(x) = x² - 2x + 3We need to find the average rate of change of the function from x = −2 to x = 5. To find this, we need to compute the slope of the line that passes through (−2, f(−2)) and (5, f(5)). Using the slope formula, we have: average rate of change = (f(5) - f(-2)) / (5 - (-2))Substitute f(5) and f(−2) into the equation, and we have: average rate of change = ((5² - 2(5) + 3) - ((-2)² - 2(-2) + 3)) / (5 - (-2))Simplify the above equation, we get: average rate of change = (28 - 7) / 7 = 3Thus, the average rate of change of the function f(x) = x² - 2x + 3 from x = −2 to x = 5 is 3.

Learn more about quadratic function here:

https://brainly.com/question/18958913

#SPJ11

Let S = {P, P1, P2, P3} and Q = {P1, P2, P3} where, p=2-1+x?; P1 =1+x, P2 = 1+r?, P3 = x +22 (a) Do the vectors of S form a linearly independent set? Show all of your work or explain your reasoning. (b) Do the vectors of Q form a linearly independent set? Show all of your work or explain your reasoning. (c) Is S a basis for P,? Recall that P, is the vector space of polynomials of degree < 2. Circle YES or NO and Explain Briefly. (d) Is Q a basis for P2? Circle YES or NO and Explain Briefly. = (e) Find the coordinate vector of p relative to the set Q = {P1, P2, P3}. That is express p as a linear combination of the vectors in S. p = 2-2 +2?; P1 =1+r, P2 = 1+x2, P3 = 1+

Answers

The only solution to the equation aP + bP1 + cP2 + dP3 = 0 is the trivial one a = b = c = d = 0. Therefore, the vectors of S form a linearly independent set.

(a) To determine whether the vectors of S form a linearly independent set, we need to check if the equation aP + bP1 + cP2 + dP3 = 0 has only the trivial solution a = b = c = d = 0.

Substituting the given vectors into the equation, we get:

a(2 - 1 + x) + b(1 + x) + c(1 + r) + d(x + 22) = 0

Simplifying, we get:

ax + bx + c + cr + dx + 2d = 0

Rearranging and grouping the terms by powers of x, we get:

x(a + b + d) + (c + cr + 2d) = 0

Since this equation must hold for all values of x, we can set x = 0 and x = 1 to get two equations:

c + cr + 2d = 0 (when x = 0)

a + b + d = 0 (when x = 1)

We can also set x = -1 to get another equation:

-2a + 2b - d = 0 (when x = -1)

Now we have a system of three equations:

c + cr + 2d = 0

a + b + d = 0

-2a + 2b - d = 0

Solving this system, we get:

a = 2d/3

b = d/3

c = -cr - 4d/3

Since c must be zero (since there is no x term in P), we get:

cr + 4d/3 = 0

If c is not zero, then the vectors of S are linearly dependent. However, since this equation holds for all r and d, we must have c = 0 as well.

Thus, the only solution to the equation aP + bP1 + cP2 + dP3 = 0 is the trivial one a = b = c = d = 0. Therefore, the vectors of S form a linearly independent set.

To know more about vectors refer here:

https://brainly.com/question/29740341

#SPJ11

find the length of x

Answers

The answer is: x= 4.8

How did I get this answer:

suppose that f (n) = f (n∕3) 1 when n is a positive integer divisible by 3, and f (1) = 1. Find a) f(3) b) f(27)c) (729)

Answers

a) f(3) = 2, b) f(27) = 4, and c) f(729) = 7.

To find f(3), we use the formula f(n) = f(n/3) + 1 when n is a positive integer divisible by 3. Since 3 is divisible by 3, we have f(3) = f(3/3) + 1 = f(1) + 1 = 1 + 1 = 2.
To find f(27), we again use the formula f(n) = f(n/3) + 1 when n is a positive integer divisible by 3. Since 27 is divisible by 3, we have f(27) = f(27/3) + 1 = f(9) + 1. To find f(9), we again apply the formula, f(9) = f(9/3) + 1 = f(3) + 1. We know that f(3) = 2, so we have f(9) = 2 + 1 = 3. Therefore, f(27) = f(9) + 1 = 3 + 1 = 4.
To find f(729), we again apply the formula, f(729) = f(729/3) + 1 = f(243) + 1. To find f(243), we again apply the formula, f(243) = f(243/3) + 1 = f(81) + 1. To find f(81), we again apply the formula, f(81) = f(81/3) + 1 = f(27) + 1. We know that f(27) = 4, so we have f(81) = 4 + 1 = 5. Therefore, f(243) = f(81) + 1 = 5 + 1 = 6. Finally, we have f(729) = f(243) + 1 = 6 + 1 = 7.
In summary, a) f(3) = 2, b) f(27) = 4, and c) f(729) = 7.

To know more about Positive Integer visit:
https://brainly.com/question/13733829
#SPJ11

find the dimensions of the box with volume 8000 cm3 that has minimal surface area. (let x, y, and z be the dimensions of the box.) (x, y, z) =

Answers

The dimensions of the box with a volume of 8000 cm³ and minimal surface area would be (20 cm, 20 cm, 20 cm), forming a cube.

Let's assume the dimensions of the box are x, y, and z. The volume of the box is given as 8000 cm³, so we have the equation:

x * y * z = 8000

To minimize the surface area, we need to minimize the sum of the areas of all six sides of the box. The surface area of a rectangular box is given by:

2xy + 2xz + 2yz

We can rewrite this equation as:

2xy + 2xz + 2yz = 2(x * y + x * z + y * z)

To minimize the surface area, we want to minimize the values of x, y, and z while still satisfying the volume constraint. The dimensions that result in the smallest surface area while maintaining the volume of 8000 cm³ are when x = y = z = 20 cm, which gives us a cube-shaped box.

Therefore, the dimensions of the box with a volume of 8000 cm³ and minimal surface area would be (20 cm, 20 cm, 20 cm), forming a cube.

Learn more about surface area here:

https://brainly.com/question/29298005

#SPJ11

Tom needs $80 to buy his dad a birthday gift. He has saved 75% of that amount so far. How much has he saved so far?

Answers

Tom has saved 75% of $80 so far to buy his dad a birthday gift.

To find out how much Tom has saved so far, we need to calculate 75% of $80. To calculate a percentage, we multiply the percentage value by the total amount. In this case, we multiply 75% (expressed as a decimal, 0.75) by $80.
0.75 * $80 = $60
Therefore, Tom has saved $60 so far, which is 75% of the total amount needed for the gift. He still needs an additional $20 ($80 - $60) to reach his goal of $80.

Learn more about saved here
https://brainly.com/question/25378221



#SPJ11

Other Questions
I need help so I'm giving 100 points away if you help with this questionThe use of grass filter _______ can help block nitrates and pesticides from washing into the water system in runoff from cultivated fieldsthere are no answer choices so please answer slowly and think about it The two pentagons seen below are similar figures. What is the scale factor from the small pentagon on the left to the largepentagon on the right? The pentagons are not drawn to scale In Act I, Scene I, of the Tragedy of Romeo and Juliet, what penalty does Prince Escalus threaten if further violence breaks out between the capulets and the montagues Pls help Ill brain-lest ASAP I nome when I met juma's brotheras walked bl have walkedc. have been walkwalking d, was walking What are the steps to finding the area of a parallelogram? What are the steps to finding the area of an isosceles trapezoid A store clerk is paid $10.75/ hour and a store assistant manager is paid $16.25/ hour. How long would it take the store managerto make $500 or more than the store clerk? Round to the nearest hour. Show all work. Alicia quiere comprar 57 vasos. En la tienda ve que los vasos se venden, en cajasde 3 o de 5 piezas. Alicia compra en total 13 cajas, de manera que pueda tenerexactamente 57 vasos.Cuntas cajas de tres vasos y cuntas cajas de cinco vasos ha compradoAlicia? True or false: is x=4 a solution to |x+5| =-9?. Explain [tex]32 \frac{1}{2} \times \sqrt[8]{16} \div (0.008) \frac{1}{3} [/tex] i need help! plz. im not to good at ratios and dont mind the one i clicked i just clicked on a random one More of my sisters work DONT ANSWER IF YOU DONT KNOWTruth says that between the Negroes of the South and the Women of the North they are all talking about their rights. Why do you think Truth chose these specific words to describe women? Why do cells continue to divide in adult organisms? I need help plz, thank you!!! 15 3/4percent is equal to which decimal Which expression is equivalent to 3 4 ( 8 y 12 ) + 1 5 ( 15 y 30 ) ? A. 3y 15 B. 3y + 3 C. 2y 12 D. y 3 what are the values of AB and DE in parallelogram ABCD? How was Dr. Martin Luther King, Jr., important to the Civil Rights movement? At least 2 complete sentences Tia earns $19.25 per hour she worked 36 hours.what is her straight time pay 5. Ronnie needs to earn more than $460 every month so he can save enough to go on vacation at the endof the year. He is paid $9.50 per hour and a monthly bonus of $60. Write an inequality that representsthis situation, where h represents the number of hours that Ronnie needs to work in a month to earnmore than $460 and meet his goal.What would be the minimum number of hours, rounded to the nearest whole, that Ronnie would haveto work?A. 40 hoursB. 41 hoursC. 42 hoursD. 43 hours